Search found 324 matches


how many?

approximately how many sentences should a paragraph contain?

by rahul.s

Wed Feb 24, 2010 9:55 pm
Forum: GMAT Essays (AWA)
Topic: how many?
Replies: 1
Views: 1739

gmat score

i know that i can request to send my gmat score to 5 schools before the test, but when can i request my score to be sent to additional schools. do i have to wait for my official score report?

by rahul.s

Wed Feb 24, 2010 9:51 pm
Forum: GMAT Strategy
Topic: gmat score
Replies: 3
Views: 1212

i looked this problem up and the OA is A, but t is a positive integer.

http://www.manhattangmat.com/forums/if- ... t4697.html

by rahul.s

Wed Feb 24, 2010 5:45 am
Forum: Data Sufficiency
Topic: hard question from GMATPrep
Replies: 7
Views: 2030

what's the OA?

for statement (1), why can't we consider t = 6?

by rahul.s

Wed Feb 24, 2010 5:43 am
Forum: Data Sufficiency
Topic: hard question from GMATPrep
Replies: 7
Views: 2030

(C) falling employment rates triggered by a drop in investment, which cause cutbacks in consumer spending, starting a cycle of layoffs that lead to even lower employment rates. yup, i think C's the correct choice. here, which is modifying neither investment, nor a drop in investment because they do...

by rahul.s

Wed Feb 24, 2010 5:14 am
Forum: Sentence Correction
Topic: Economic recession and lower employment rates
Replies: 19
Views: 6884

i looked it up and komal's problem includes the correct answer choices. the choices on the mgmat forum are incorrect http://gmatclub.com/forum/an-economic-recession-81080.html here, i think "which" correctly modifies "rates" ; the word "which" modifies the noun immediat...

by rahul.s

Wed Feb 24, 2010 4:21 am
Forum: Sentence Correction
Topic: Economic recession and lower employment rates
Replies: 19
Views: 6884

i came across the same problem, but there seems to be a disparity between the answer choices. the OA is C but answer choice C is: c. falling employment rates triggered by a drop in investment, causing cutbacks in consumer spending and starting a cycle of layoffs that lead to even lower employment ra...

by rahul.s

Wed Feb 24, 2010 4:15 am
Forum: Sentence Correction
Topic: Economic recession and lower employment rates
Replies: 19
Views: 6884

(C) falling employment rates triggered by a drop in investment, which cause cutbacks in consumer spending, starting a cycle of layoffs that lead to even lower employment rates. "which" modifies the noun immediately preceding it. in this sentence, "investment, which cause ." this...

by rahul.s

Wed Feb 24, 2010 4:11 am
Forum: Sentence Correction
Topic: Economic recession and lower employment rates
Replies: 19
Views: 6884

yeah, i second phoenix's opinion. start with the diagnostic from the OG.

by rahul.s

Wed Feb 24, 2010 3:22 am
Forum: GMAT Strategy
Topic: Diagnostic test
Replies: 3
Views: 1207

isvas wrote:Can I take GMAT prep CATs more than once/
Do a lot of Qs repeat themselves?
I took the GMATPrep 1 twice, but came across only a couple of problems that I had encountered earlier. It's definitely worth taking twice.

by rahul.s

Wed Feb 24, 2010 3:20 am
Forum: GMAT Strategy
Topic: GMAT Prep CATs multiple times
Replies: 4
Views: 1378

This is a problem from the OG, and it's been discussed earlier.

http://www.beatthegmat.com/square-count ... 21427.html

by rahul.s

Tue Feb 23, 2010 10:49 pm
Forum: Problem Solving
Topic: Very tough Math problem!!
Replies: 10
Views: 2320

Hey u will bang rite on the answer choice if you use the rogue element approach. Look carefully at the premise- the glass separates green and blue light from white light and the conclusion says historians want to give Alhazen the title of father of modern optics for discovering dispersive prism. No...

by rahul.s

Tue Feb 23, 2010 8:17 am
Forum: Critical Reasoning
Topic: a broken shard of glass
Replies: 4
Views: 2210

John Mercer Brooke created a sounding apparatus that is essentially a rod, connected to a string, that strikes the sea floor, causing the tension on the string to slack and a ball to fall to the ground; the rod, released from its weight, is easily drawn up, bringing with it a cup-like apparatus, in...

by rahul.s

Tue Feb 23, 2010 7:08 am
Forum: Sentence Correction
Topic: john mercer brooke
Replies: 5
Views: 1237

yeah, harshavardhanc is right. the OE mentioned that answer choice (C) brings in no new info. i opted for (C) because i thought that reinforcing the occurrence of the tornado in 1976 would strengthen the hypothesis and that's why i asked whether we can allow for new / additional info in such problems.

by rahul.s

Tue Feb 23, 2010 7:06 am
Forum: Critical Reasoning
Topic: explorers of the abandoned city of ambrose, kansas
Replies: 16
Views: 5120